Question and Answers Forum

All Questions      Topic List

Limits Questions

Previous in All Question      Next in All Question      

Previous in Limits      Next in Limits      

Question Number 134875 by bemath last updated on 08/Mar/21

Commented by bemath last updated on 08/Mar/21

The figure shows a point P on the  parabola y=x^2  and the point Q  where the perpendicular bisector  of OP intersects the y−axis . As   P approaches the origin along  parabola , what happens to Q?  Does it have a limiting position?  If so find it.

$$\mathrm{The}\:\mathrm{figure}\:\mathrm{shows}\:\mathrm{a}\:\mathrm{point}\:\mathrm{P}\:\mathrm{on}\:\mathrm{the} \\ $$$$\mathrm{parabola}\:\mathrm{y}=\mathrm{x}^{\mathrm{2}} \:\mathrm{and}\:\mathrm{the}\:\mathrm{point}\:\mathrm{Q} \\ $$$$\mathrm{where}\:\mathrm{the}\:\mathrm{perpendicular}\:\mathrm{bisector} \\ $$$$\mathrm{of}\:\mathrm{OP}\:\mathrm{intersects}\:\mathrm{the}\:\mathrm{y}−\mathrm{axis}\:.\:\mathrm{As}\: \\ $$$$\mathrm{P}\:\mathrm{approaches}\:\mathrm{the}\:\mathrm{origin}\:\mathrm{along} \\ $$$$\mathrm{parabola}\:,\:\mathrm{what}\:\mathrm{happens}\:\mathrm{to}\:\mathrm{Q}? \\ $$$$\mathrm{Does}\:\mathrm{it}\:\mathrm{have}\:\mathrm{a}\:\mathrm{limiting}\:\mathrm{position}? \\ $$$$\mathrm{If}\:\mathrm{so}\:\mathrm{find}\:\mathrm{it}. \\ $$

Answered by mr W last updated on 08/Mar/21

P(p,p^2 )  M=midpoint OP  m_(OP) =(p^2 /p)=p  m_(MQ) =−(1/p)  M((p/2),(p^2 /2))  eqn. of MQ:  y=(p^2 /2)−(1/p)(x−(p/2))  y_Q =(p^2 /2)−(1/p)(0−(p/2))=(p^2 /2)+(1/2)  lim_(p→0) y_Q =(1/2)

$${P}\left({p},{p}^{\mathrm{2}} \right) \\ $$$${M}={midpoint}\:{OP} \\ $$$${m}_{{OP}} =\frac{{p}^{\mathrm{2}} }{{p}}={p} \\ $$$${m}_{{MQ}} =−\frac{\mathrm{1}}{{p}} \\ $$$${M}\left(\frac{{p}}{\mathrm{2}},\frac{{p}^{\mathrm{2}} }{\mathrm{2}}\right) \\ $$$${eqn}.\:{of}\:{MQ}: \\ $$$${y}=\frac{{p}^{\mathrm{2}} }{\mathrm{2}}−\frac{\mathrm{1}}{{p}}\left({x}−\frac{{p}}{\mathrm{2}}\right) \\ $$$${y}_{{Q}} =\frac{{p}^{\mathrm{2}} }{\mathrm{2}}−\frac{\mathrm{1}}{{p}}\left(\mathrm{0}−\frac{{p}}{\mathrm{2}}\right)=\frac{{p}^{\mathrm{2}} }{\mathrm{2}}+\frac{\mathrm{1}}{\mathrm{2}} \\ $$$$\underset{{p}\rightarrow\mathrm{0}} {\mathrm{lim}}{y}_{{Q}} =\frac{\mathrm{1}}{\mathrm{2}} \\ $$

Terms of Service

Privacy Policy

Contact: info@tinkutara.com